Epsilon-delta proof for limits (multivariable)

Click For Summary
The discussion revolves around proving the limit of the function f(x,y) as x approaches 1.3 and y approaches -1, with the expected limit being (3.3, 4.4, 0.3). A participant expresses confusion regarding the choice of 11 in the delta-epsilon proof, questioning its derivation and whether it can be replaced with other values like 10 or 12. Responses clarify that the specific number is often based on experience and that such prefactors are generally flexible, with alternatives being acceptable as long as they fit within the proof's context. The importance of clarity in notation, particularly in mathematical expressions, is also emphasized. Understanding the reasoning behind these choices is crucial for mastering epsilon-delta proofs in multivariable calculus.
Zeeree
Messages
5
Reaction score
0

Homework Statement

:
the question wants me to prove that the limit of f(x,y) as x approaches 1.3 and y approaches -1 is (3.3, 4.4, 0.3). f(x,y) is defined as (2y2+x, -2x+7, x+y).
[/B]

The attempt at a solution: This is the solution my lecturer has given. it's not very neat, sorry.
http://imgur.com/CfmIodw

So far, I've managed to understand what's going on, up till the part that says choose delta as min{1, epsilon/11} I'm really not sure how he got the 11 over there in the denominator. I can see how it fits at the last line of the page and it makes sense. but How did he get it firstly? Is it arbitrary, can it also be something like 10, 12 etc?

Sorry if this seems silly.
 
Physics news on Phys.org
##\displaystyle \frac \epsilon {12}## would work as well. 10 as denominator gives a value that is too large.

How to get 11: experience. If in doubt, take 1000. Or leave the denominator blank and fill it out once you got to the last line.
Such a prefactor is irrelevant - you'll also often see proofs that end with ##|a-b| < 11 \epsilon## instead of having the 11 earlier. That is perfectly fine.
 
Zeeree said:

Homework Statement

:[/B]
the question wants me to prove that the limit of f(x,y) as x approaches 1.3 and y approaches -1 is (3.3, 4.4, 0.3).
f(x,y) is defined as (2y2+x, -2x+7, x+y).The attempt at a solution: This is the solution my lecturer has given. it's not very neat, sorry.
http://imgur.com/CfmIodw

So far, I've managed to understand what's going on, up till the part that says choose delta as min{1, epsilon/11} I'm really not sure how he got the 11 over there in the denominator. I can see how it fits at the last line of the page and it makes sense. but How did he get it firstly? Is it arbitrary, can it also be something like 10, 12 etc?

Sorry if this seems silly.[/B]
Hello Zeeree. Welcome to PF !

I see that the function is given as
upload_2016-7-12_21-31-52.png


At the very least, you should use ^ to indicate exponentiation. Even better, use the X2 feature or LaTeX.

f(x,y) = (2y2+x, -2x+7, x+y) .
 
Question: A clock's minute hand has length 4 and its hour hand has length 3. What is the distance between the tips at the moment when it is increasing most rapidly?(Putnam Exam Question) Answer: Making assumption that both the hands moves at constant angular velocities, the answer is ## \sqrt{7} .## But don't you think this assumption is somewhat doubtful and wrong?

Similar threads

  • · Replies 31 ·
2
Replies
31
Views
2K
  • · Replies 9 ·
Replies
9
Views
4K
  • · Replies 6 ·
Replies
6
Views
2K
  • · Replies 5 ·
Replies
5
Views
5K
  • · Replies 15 ·
Replies
15
Views
6K
  • · Replies 5 ·
Replies
5
Views
2K
  • · Replies 8 ·
Replies
8
Views
2K
Replies
19
Views
2K
  • · Replies 12 ·
Replies
12
Views
3K
Replies
20
Views
2K